You are on page 1of 17

MỘT SỐ VẤN ĐỀ VỀ

ĐA THỨC BẤT KHẢ QUY

Lê Phúc Lữ 1
2
Nguyễn Tuấn Anh

Trong chủ đề này, chúng ta sẽ cùng hệ thống lại các kiến thức về tính khả quy, bất khả
quy của đa thức cũng như một số tiêu chuẩn để chứng minh đa thức bất khả quy trên
tập hợp số nguyên. Các bài toán về việc khảo sát tính bất khả quy của đa thức một
hay nhiều biến luôn khá thú vị. Nó có liên hệ đến các biến đổi đại số, số học cũng
như đánh giá module của các nghiệm phức của đa thức. Với các tiêu chuẩn đủ mạnh,
nhiều bài toán dạng này sẽ được giải quyết một cách khá nhanh chóng.

1. Các kiến thức cần nhớ.


1.1. Đa thức bất khả quy.
n
Định nghĩa: Đa thức P(x) =
P
ai x i có các hệ số thuộc tập hợp K được gọi là bất khả
i=0
quy trên K nếu P(x) không thể biểu diễn được dưới dạng tích của hai đa thức có hệ số
thuộc K và có bậc không nhỏ hơn 1.
Rõ ràng nếu P(x) bất khả quy thì P(ax + b) với a, b ∈ Z cũng thế. Từ đây, suy ra mọi
đa thức có bậc không vượt quá 1 đều bất khả quy.
Tính khả quy trên C : Ta biết rằng mỗi đa thức bậc n > 1 đều có n nghiệm và theo
định lý Bezout thì nếu P(x) có nghiệm là x = α thì tồn tại đa thức Q(x) sao cho
P(x) = (x − α)Q(x) nên mọi đa thức có bậc lớn hơn 1 đều khả quy trên C.
1
Trường Đại học FPT thành phố Hồ Chí Minh
2
Trường THPT Chuyên KHTN, ĐHQG Hà Nội

1
2

Tính khả quy trên R : Ta sẽ chứng minh rằng mọi đa thức có hệ số thực và bậc lớn
hơn 2 đều khả quy trên R.
Thật vậy, dễ thấy rằng các đa thức bậc hai vô nghiệm trên R thì bất khả quy. Xét một
đa thức P(x) có deg P > 2. Ta có 2 trường hợp:
• Nếu P(x) có nghiệm thực là x = α thì tương tự trường hợp trên, ta thấy P(x)
khả quy.

• Nếu P(x) vô nghiệm, rõ ràng với mỗi nghiệm phức x = α thì x = ᾱ cũng là
nghiệm của P(x). Dễ thấy Q(x) = (x − α)(x − ᾱ) = x 2 − Re(α) + |α|2 là đa thức
có hệ số thực và khi đó, tồn tại R(x) sao cho P(x) = Q(x)R(x), tức là P(x) khả
quy trên R.
Tóm lại ta không quan tâm đến tính khả quy của đa thức trên C, R. Trong các nội
dung dưới đây, hầu hết ta xét tính khả quy (bất khả quy) trên Z, Q và tương ứng
trong đó, nếu không nói gì thêm, ta sẽ quy ước xét các đa thức hệ số nguyên.
1.2. Tính chất số học của đa thức có hệ số nguyên.
Cho hai đa thức P(x), Q(x) sao cho không tồn tại đa thức R(x) nào thỏa mãn 1 ¶
deg R(x) ¶ min{P(x), Q(x)} và P(x), Q(x) đều chia hết cho R(x) thì ta nói hai đa
thức P(x), Q(x) nguyên tố cùng nhau.
Hai đa thức P(x), Q(x) khi chia cho đa thức f (x) được cùng một đa thức dư thì ta
nói hai đa thức này đồng dư với nhau theo modulo f (x).
Định lý Bezout: Với hai đa thức P(x), Q(x) nguyên tố cùng nhau, tồn tại hai đa thức
A(x), B(x) sao cho P(x)A(x) + Q(x)B(x) = 1.
n
p
Nếu đa thức P(x) =
P
ai x i có hệ số nguyên và có nghiệm hữu tỉ là q thì p là ước của
i=0
a0 và q là ước của an .
Đa thức monic là đa thức có hệ số bậc cao nhất là 1, từ đó dễ thấy rằng, nếu nó có
nghiệm hữu tỉ thì nghiệm đó cũng nguyên.
1.3. Tiêu chuẩn Eisenstein và dạng mở rộng.
Dưới đây là tiêu chuẩn khá phổ biến để chứng minh một đa thức là bất khả quy:
n
Tiêu chuẩn Eisenstein: Cho đa thức P(x) = ai x i với ai ∈ Z, i = 0, n. Khi đó, nếu
P
i=0
tồn tại số nguyên tố p thỏa mãn đồng thời các điều kiện:
• Tất cả các hệ số, trừ hệ số đầu tiên là an , đều chia hết cho p.

• Hệ số an không chia hết cho p2 .


Khi đó, đa thức P(x) bất khả quy trên tập hợp Z.
Chứng minh: Giả sử ngược lại, tồn tại f , g là các đa thức có hệ số nguyên khác hằng
r s
sao cho P(x) = f (x)g(x) với f (x) = bi x i , g(x) = ci x i và r + s = n. Dễ thấy
P P
i=0 i=0
. .
rằng a0 = b0 c0 , mà a0 ..p và a0 6 ..p2 nên trong hai số b0 , c0 có đúng 1 số chia hết cho p,
giả sử là b0 . Ngoài ra, do an = b r cs không chia hết cho p nên b r không chia hết cho
p. Suy ra, tồn tại k là số lớn nhất mà 0 < k ¶ r < n sao cho bk không chia hết cho p.
Khi đó b0 , b1 , b2 , . . . , bk−1 đều chia hết cho p. Ta cũng có

ak = bk c0 + bk−1 c1 + · · · + b0 ck ,
3

. .
là hệ số chia hết cho p. Tuy nhiên, do bk c0 6 ..p và b0 , b1 , . . . , bk−1 ..p nên đây là điều
mâu thuẫn.
Vậy điều giả sử là sai và ta có đpcm.

Ta thử áp dụng tiêu chuẩn này để giải quyết một bài toán THCS như sau:
Ví dụ. Chứng minh đa thức P(x) = x 4 − x 3 + 2x + 1 bất khả quy.
Chúng ta hãy xem xét và thử so sánh 2 cách giải sau:
Cách 1: Do đây là đa thức monic nên nếu nó có nghiệm hữu tỉ thì nghiệm đó phải
nguyên. Ngoài ra, hệ số tự do là 1 nên nghiệm nguyên của P(x) chỉ có thể là ±1,
thay vào ta thấy không thỏa. Suy ra P(x) không có nghiệm nguyên nên nếu P(x)
phân tích được thành nhân tử thì đó phải là tích của hai tam thức bậc hai, cụ thể là
P(x) = (x 2 + ax + b)(x 2 + c x + d), với mọi a, b, c, d ∈ Q. Đồng nhất hệ số, ta được

a + c = −1

 b + d + ac = 0

 ad + bc = 2

bd = 1

/ Q nên đa thức đã cho bất khả quy trên Q).


Giải hệ này ra, ta thấy a, b, c, d ∈
Cách 2: Đặt Q(x) = P(x + 1) = (x + 1)4 − (x + 1)3 + 2(x + 1) + 1 = x 4 + 3x 3 + 3x 2 +
3x + 3, khi đó theo tiêu chuẩn Eisenstein, dễ thấy đa thức Q(x) bất khả quy nên P(x)
cũng thế.
Ta thấy cách chứng minh thứ 2 khá ngắn gọn và ấn tượng. Ta sẽ đề cập tiếp đến việc
đổi biến để áp dụng được tiêu chuẩn Eisenstein trong các ví dụ áp dụng sau. Dưới đây
là một mở rộng của tiêu chuẩn này, ý tưởng chứng minh tương tự.
n
Tiêu chuẩn Eisenstein suy rộng: Cho P(x) =
P
ai x i là một đa thức hệ số nguyên.
i=0
Nếu tồn tại số nguyên tố p thỏa mãn những điều kiện sau:

• an không chia hết cho p.

• Các hệ số a0 , a1 , . . . , an−k chia hết cho p.

• a0 không chia hết cho p2 .

Đa thức P(x) có ước không phân tích được là G(x), mà bậc của đa thức này lớn hơn
hoặc bằng k. Bạn đọc thử tự chứng minh tiêu chí này bằng lập luận tương tự trên.
Tiếp theo, ta có một bổ đề khá đơn giản nhưng có thể làm rút gọn công việc lập luận
khá nhiều khi xử lý các bài toán về đa thức bất khả quy.
Bổ đề Gauss: Đa thức P(x) bất khả quy trên Z khi và chỉ khi nó bất khả quy trên Q.
Khi đó, rõ ràng ta có thể linh hoạt chuyển đổi việc lập luận trên tập Z hay tập Q khi
làm việc với các bài toán về đa thức bất khả quy.
Tiêu chí trên cho phép ta đánh giá đa thức bất khả quy khi xem xét các tính chất số học
của các hệ số. Hai tiêu chí dưới đây còn cho ta phép các quan hệ so sánh đại số thông
thường giữa các hệ số này. Tuy có sử dụng đến kiến thức về số phức
nhưng chỉ dừng ở
các kết quả quen thuộc như: bất đẳng thức về module |x| + y ¾ x + y ¾ |x| − y ,

đa thức bậc n thì có n nghiệm thực hoặc phức.
4

1.4. Tiêu chuẩn Perron:


n
Cho đa thức nguyên P(x) = ai x i có a0 6= 0. Khi đó, nếu
P
i=0


a > a + a + · · · + a + a ,
n−1 0 1 n−2 n

thì đa thức này bất khả quy.

Chứng minh. Không mất tính tổng quát,


P(x)
ta
có thể
giả an = 1 (vì P(x) bất khả quy
sử
khi và chỉ khi a bất khả quy). Ta có an−1 > a0 + a1 + ... + an−2 + 1. Ta sẽ chứng

n

minh rằng tồn tại đúng một nghiệm (thực hoặc phức) của P(x) có module lớn hơn 1.
Thật vậy,giả sử |z| thì

a = a z n−1 = z n + a z n−2 + · · · + a z + a ¶ 1 + a + a + · · · + a ,
n−1 n−1 n−2 1 0 0 1 n−2


mâu thuẫn. Ngoài ra, f (0) = a0 6= 0 nên f (0) ¾ 1 là tích các module tất cả các
nghiệm của P(x) nên tồn tại một nghiệm x 1 sao cho x 1 > 1. Gọi x 2 , x 3 , . . . , x n là
các nghiệm còn lại. Đặt

f (x)
g(x) = x n−1 + bn−2 x n−2 + · · · + b1 x + b0 = ,
x − x1

đồng nhất hệ số (x − x 1 )g(x) = f (x) thì

an−1 = bn−2 − x 1 , an−2 = bn−3 − bn−2 x 1 , . . . , a1 = b0 − b1 x 1 , a0 = −b0 x 1 .



Theo giả thiết thì bn−2 − x 1 > 1 + bn−3 − bn−2 x 1 + · · · + b0 x 1 , suy ra

b + x > 1 + b − b x + b − b x + · · · + b x ,
n−2 1 n−3 n−2 1 n−4 n−3 1 0 1

hay € Š € Š
x − 1 > x − 1 b + b + ... + b
1 1 0 1 n−2

Do x 1 > 1 nên b0 + b1 + · · · + bn−2 < 1.

Rõ ràng nghiệm (thực hoặc phức) của g(x) chỉ có module nhỏ hơn 1 vì nếu không
giả sử nó có nghiệm z sao cho |z| > 1 thì

bn−2 b1 b0
1+ + ··· + n−1
+ = 0,
z z zn
suy ra
bn−2 b1 b0
b + b + · · · + b ¾
z + · · · + z n−1 + z n = 1,

0 1 n−2

ddiều mâu thuẫn này cho thấy f (x) chỉ có đúng 1 nghiệm có module lớn hơn 1.
Giả sử rằng f (x) khả quy, tức là tồn tại hai đa thức có hệ số nguyên P(x), Q(x)
bậc không nhỏ hơn 1 sao cho, trong đó, tất cả các nghiệm của P(x) đều có module
nhỏ hơn 1. Suy ra |P(0)| < 1 vì |P(0)| là tích các module của nghiệm của P(x), mà
P(0) ∈ Z nên P(0) = 0, dẫn đến f (0) = 0, mâu thuẫn. Vậy đa thức f (x) bất khả quy.
5

Tiếp theo là một tiêu chuẩn tương tự:


1.4. Tiêu chuẩn về đánh giá nghiệm:
Cho đa thức P(x) có hệ số nguyên và đặt m ∈ Z thỏa mãn |P(m)| là 1 hoặc là một
số nguyên tố sao cho P(x) không có nghiệm thuộc {z ∈ C : |z − m| ¶ 1}. Khi đó, P(x)
bất khả quy.

Chứng minh. Giả sử tồn tại đa thức Q(x), R(x) thỏa mãn P(x) = Q(x)R(x). Theo
giả thiết thì |Q(m)| = 1 hoặc |R(m)| = 1. Không mất tính tổng quát, ta có thể giả sử
r
|R(m)| = 1. Ta có thể giả sử Q(x) = a (x − αi ) với a ∈ Z, deg Q(x) = r ¾ 1 và
Q
i=1
r
αi , i = 1, n là các nghiệm của đa thức Q(x). Khi đó, ta có 1 = |Q(m)| = |a| m − αi .
Q
i=1
Do a 6= 0 nên |a| ¾ 1 và tồn tại sô nguyên dương j ∈ {1, 2, ..., r} thỏa mãn m − αi ¶ 1
và P(αi ) = Q(αi )R(αi ) = 0. Điều này là vô lý do nghiệm z của P(x) đều thỏa mãn
|z − m| > 1. Do đó, điều giả sử là sai và đa thức đã cho bất khả quy.

Nhờ tiêu chuẩn này, ta có thể chứng minh nhóng chóng các bài toán khó sau:
Bài 1. Chứng minh rằng với mọi n ¾ 2 thì đa thức x n + 29x n−1 + 2009 bất khả quy.
Bài 2. Chứng minh rằng với mọi n ¾ 2 thì đa thức x n + 5x n−1 + 3 bất khả quy.

Bài 3. Cho đa thức f (x) = x n +an−1 x n−1 +· · ·+a1 x +a0 thỏa mãn a0 > a1 + a2 +· · ·+ an
và a0 là số nguyên tố thì f (x) bất khả quy.
n
a
Bài 4. Cho đa thức hệ số nguyên f (x) = ai x i và đặt G = max a i . Khi đó, nếu tồn tại m
P
n
i=0
sao cho m ¾ G + 2 và f (m) nguyên tố thì đa thức f (x) bất khả quy.

(Tạp chí AMM)

Trong các ví dụ ở phần 2, ta sẽ cùng tìm hiểu thêm một số bài toán áp dụng hai tiêu chuẩn
này để xử lý.

2. Các ví dụ áp dụng.
Ví dụ 1. Sử dụng tiêu chuẩn Eisenstein để chứng minh các đa thức sau bất khả quy.

1) f (x) = x 4 + x 3 + x 2 + 6x + 1.

2) Đa thức x n − a với a là số nguyên dương mà trong phân tích thành thừa số nguyên
tố có một ước nguyên tố có số mũ là 1.
n
3) Đa thức f (x) = ai x i với p là số nguyên tố và n = p k n0 với gcd(p, n0 ) = 1,
P
i=0
đồng thời vp (an ) = 0, vp (a0 ) = n − 1 và vp (ai ) ¾ n − i, i = 1, n − 1.

Lời giải. 1) Ta có g(x) = f (x + 1) = y 4 + 5 y 3 + 10 y 2 + 15 y + 10. Theo tiêu chuẩn


Eisensteins thì đa thức này bất khả quy.
2) Gọi p là số nguyên tố duy nhất mà đề bài nhắc đến. Chú ý rằng 0 và a đều chia hết
cho p và a lại không chia hết cho p2 .
1
€pŠ
3) Đặt g(x) = pn−1 x n f x thì có thể áp dụng trực tiếp được tiêu chuẩn Eisenstein.
6

Chú ý.
Ta đặt ra câu hỏi sau một cách tự nhiên: Có tồn tại đa thức bất khả quy nào mà nó không có
một cách biến đổi tuyến tính nào đó của ẩn thể chuyển nó thành đa thức có thể áp dụng được
tiêu chuẩn Eisenstein (với số nguyên tố p). Câu trả lời là khẳng định.
Thật vậy, xét đa thức P(x) = 2x 2 + 1 bất khả quy trên Z. Đặt x = a x + b với a, b là những
số nguyên bất kì. Ta có Q( y) = 2a2 y 2 + 4a b y + 2b2 + 1. Giả sử Q(x) là đa thức có dạng
. .
Eisenstein thì tồn tại số nguyên tố sao cho 2a2 không chia hết cho p, 4a b .. p, (2b2 + 1) .. p
. .
nên b không chia hết pnhưng khi đó, ta có 4p |a Do đó, a .. p hoặc là 4 .. p. Ta chỉ cần xét 2
trường hợp:
.
1. Nếu deg Q( y) .. p trái với giả thiết.
.
2. Nếu 4 .. p suy ra p = 2 nhưng khi đó, đa thức (2b2 + 1) lại không chia hết cho p.

Điều mâu thuẫn này cho thấy nhận xét ban đầu là đúng.
Ví dụ 2.

1) Cho các đa thức nguyên f1 (x), f2 (x), ..., f n (x) bất khả quy. Chứng minh rằng tồn
tại một đa thức khả quy g(x) sao cho các đa thức sau đều bất khả quy f1 (x) +
g(x), f2 (x) + g(x), ..., f n (x) + g(x).

(Iran TST 2003)

2) Xét dãy Fibonacci: F0 = F1 = 1, Fn+1 = Fn + Fn−1 và đặt an = Fn Fn+1 . Chứng minh


rằng đa thức P(x) = x n + an−1 x n−1 + · · · + a1 x + a0 bất khả quy.

(Mathlinks contest lần 4, vòng 7)

n
3) Cho đa thức P(x) = (2k + 1)x k . Chứng minh rằng nếu 2n + 1 là số nguyên tố
P
k=0
thì đa thức này bất khả quy.

Lời giải. 1) Áp dụng tiêu chuẩn Perron. Ta chỉ cần chọn g(x) = x m + M x m−1 là một
đa thức khả quy và hệ số M đủ lớn để thỏa mãn điều kiện bất đẳng thức là xong.
2) Theo tiêu chuẩn Perron, ta chỉ cần chứng minh Fn+1 Fn > Fn Fn−1 + · · · + F2 F1 + 1.
Tuy nhiên, bất đẳng thức này có thể chứng minh dễ dàng bằng quy nạp.
3) Để giải quyết bài toán này, ta cần tiêu chuẩn thứ 4 ở phần I và bổ đề sau: Cho
n
đa thức P(x) = ai x i có hệ số thực và ai > 0 với mọi i = 0, 1, 2, ..., n. Khi đó, mọi
P
i=0
nghiệm (thực hoặc phức) của P(x) đều thỏa mãn

ai ai
   
min ¶ |z| ¶ max
0¶i<n ai+1 0¶i<n ai+1
€ Š
Trở lại bài toán, đặt Q(x) = x n P 1x = x n + 3x n−1 + · · · + 2n + 1 là một đa thức.

Theo bổ đề trên thì tất có các nghiệm z của Q(x) thỏa mãn |z| > 1, áp dụng tiêu
chuẩn 4 với m = 0, đa thức Q(x) thỏa mãn điều kiện đặt ra nên Q(x) bất khả quy. Ta
có điều phải chứng minh.
7

Chú ý.
Bài toán tổng quát với 2n + 1 là số nguyên tùy ý vẫn là câu hỏi mở.
(x+1)n −x n −1
Có nhiều bài toán mở kiểu như vậy, chẳng hạn như: Cho đa thức f (x) = x
với n ¾ 1.
Chứng minh rằng f (x) bất khả quy.
Người ta chỉ mới xử lý được trong trường hợp n = 2p với p là số nguyên
€ Š tố. Bạn đọc có thể
thử sức bằng gợi ý: Nếu f (x) = g(x)h(x) thì phải có g(x) = x n−1 h 1x . Cũng áp dụng bổ đề
Perron, ta có thể giải quyết được dễ dàng bài toán trong đề chọn đội tuyển Romani 2010.
Cho p là số nguyên tố và n1 , n2 , ..., n p là các số tự nhiên. Chứng minh rằng đa thức

x n1 + x n2 + ... + x np − p
f (x) =
xn − 1

với n = gcd(n1 , n2 , ..., n p ) bất khả quy.

Ví dụ 3. Tìm tất cả các số nguyên dương k sao cho có vô số số nguyên dương n mà đa


thức sau P(x) = x n+1 + kx n − 870x 2 + 1945x + 1995, khả quy trên tập hợp Z.

(Việt Nam TST 1995)

Lời giải. Với k là một số thỏa mãn đề bài, đặt A là tập hợp các số nguyên dương n sao
cho P(x) khả quy. Giả sử P(x) = f (x)g(x) thì theo tiêu chuẩn Eisenteisn suy rộng, ta
có deg f (x) = 1 hoặc deg g(x) = 1.
Từ đó suy ra P(x) có nghiệm nguyên là x 0 (dễ thấy x 0 6= 0). Ta xét các trường hợp:

• Nếu x 0 ¾ 2 với mọi n ∈ A thì dễ thấy rằng P(x 0 ) = 0 sẽ tương đương

870x 02 − 1945x 0 − 1995 5 (x 0 − 3)(174x 0 + 133)
x + k = = .
0 n n
x x
0 0

5|(x 0 −3)(174x 0 +133)|


Do x 0 ¾ 2, và A có vô số phần tử nên với n đủ lớn, ta có n <1
| x0|
hay x 0 + k < 1 ⇒ x 0 + k = 0. Từ đây, dễ dàng có k = −3.

• Nếu x 0 = ±1, ta lại xét tiếp:

◦ Với x 0 = 1, ta được P(1) = 0 ⇒ k = −3071.


◦ Với x 0 = −1, ta được P(−1) = 0 hay (−1)n+1 + k(−1)n + 820 = 0.

Suy ra với n lẻ thì ta có k = −819 và n chẵn thì k = 821.

Thử lại ta thấy thỏa. Vậy tất cả các giá trị k cần tìm là −3701, −819, −3, 821.

Chú ý.
Việc tìm điều kiện của các hệ số để có đa thức khả quy cũng là các bài toán phổ biến. Người
ta đã nghiên cứu các đa thức dạngx n + a, x n + a x m + b, x n + a x m + b x p + c.
Chẳng hạn dưới đây là các bài toán dạng này:
.
Bài 1. Cho đa thức P(x) = x m + x n + 1 trong đó deg P(x) = m ¾ 3. Giả sử mn − 2..3. Chứng
minh rằng P(x) là đa thức khả quy.
8

Bài 2. Cho đa thức P(x) = x n + 4 với n nguyên dương. Chứng minh rằng n chia hết cho 4 khi
và chỉ khi đa thức này khả quy.

Bài 3. (Cho tam thức P(x) = x n + qx p + r với q > 1 + |r|n−1 và n chia hết cho số nguyên
dương h > 1 nào đó và |r| không là một lũy thừa bậc h. Khi đó, P(x) bất khả quy.
Bài 4. Cho n > m > 1 là các số nguyên dương lẻ. Chứng minh rằng khi đó đa thức f (x) =
x n + x m + x + 1 bất khả quy.

(China TST 2008)

Bài đầu tiên khá nhẹ nhàng khi chú ý phân tích x 3 − 1 = (x − 1)(x 2 + x + 1) dẫn đến x 3k − 1
có các ước là x 2 + x + 1. Với bài thứ 2, ta giải quyết điều kiện cần như sau (điều kiện đủ là dễ
thấy vì nếu đặt n = 4m, ta có x 4m + 4 = (x 2m − 2x m + 2)(x 2m + 2x m + 2)). Các bài 3, 4 khó
hơn và bạn đọc có thể tham khảo thêm ở [15], [16].
Giả sử rằng x n + 4 = f (x)g(x) với f , g là 2 đa thức nguyên, monic và có bậc
không nhỏ hơn
1. Gọi α1 , α2 , ..., αn là các nghiệm của x n + 4 = 0 thì |P(0)| = α1 α2 ...αn = f (0) · g(0) .

Đặt deg f = k, deg g = h với k, h ¾ 1 thì f (0) , g(0) là tích các modulo của các nghiệm
của P(x). Do các nghiệm của x n + 4 = 0 trên tập số phức là căn bậc n của 4 nên hệ số tự
do của f , g chỉ có thể là ±2 chứ không thể là ±1. Từ đó suy ra f (0) , g(0) lần lượt bằng
2k 2h
2 n = 2 n = 2 hay k = h = 2n . Nếu k lẻ thì rõ ràng f phải có nghiệm thực, đây là điều vô lý vì
x 2k + 4 > 0, ∀x ∈ R nên suy ra k là số chẵn. Do đó, n chia hết cho 4.
Ngoài ra, trong các nội dung có tại [15], [16], người ta đã chứng minh được các bài toán quen
thuộc như:
Bài 5. Đa thức x n + x + 1 bất khả quy với mọi n chia 3 dư 2.
Bài 6. Đa thức x n − x − 1 bất khả quy với mọi n. Chứng minh định lý này cũng là một công
việc không dễ dàng khi ta phải dùng đánh giá nghiệm của
Š đa thức tương tự ở bài 2) ở trên,
1 1
€
cụ thể là nếu z là nghiệm của x − x − 1 thì 2 Re z − z > 2 − 1 và đánh giá đại lượng
n
|z|

1 1 1
 
Re z1 − + z2 − + ... + zn − ,
z1 z2 zn

theo 2 cách (trong đó z1 , z2 , ..., zn là các nghiệm của x n − x − 1 = 0).

(Định lý Selmer)

Ví dụ 4. Cho đa thức f (x) có hệ số cao nhất là 1, bất khả quy và f (0) không phải là
số chính phương. Chứng minh rằng g(x) = f (x 2 ) cũng là đa thức bất khả quy.

(Romanian TST 2003)

Lời giải. Giả sử ta có phân tích g(x) = f (x 2 ) = p(x)·q(x) với f , g là 2 đa thức monic
có hệ số nguyên và bậc không nhỏhơn 1. Gọi α là một nghiệm (thực hoặc phức) của
p p p
f (x) thì p α · q α =g α = f (α) = 0. Không mất tính tổng quát, có thể giả
p
sử p α = 0.
n n
Đặt p(x) = ai x i với ai ∈ Z và n = deg p thì ai αi = 0. Từ đó, dễ thấy rằng tồn tại
P P
i=0
pi=0
các đa thức có hệ số nguyên thỏa mãn t(α)+ α·u(α) = 0, trong đó deg t, deg u < 2n.

Do đa thức u(x) khác hằng và g(x) bất khả quy nên không tồn tại đa thức khác hằng
nào chia hết cả 2 đa thức g(x), u(x). Từ đây, suy ra cũng không tồn tại đa thức khác
9

hằng nào chia hết cả 2 đa thức f (x), u(x) nên theo định lý Bezout cho đa thức, tồn
tại hai đa thức có hệ số nguyên là s(x), r(x) sao cho

s(x)u(x) + r(x) f (x) = 1.

Suy ra s(α)u(α) = 1 nên


p
α = −t(α)s(α) ⇒ α2 = t 2 (α)s2 (α).

Đặt m(x) = t 2 (x)s2 (x) − x thì m(x) là đa thức có hệ số nguyên và m(α) = 0 , suy ra
đa thức m(x) chia hết cho đa thức f (x).
Đặt α1 , α2 , ..., αn là các nghiệm của đa thức f (x) thì m(αi ) = 0, i = 1, n và

αi = t 2 (αi )s2 (αi ).

Ta đưa về
α1 α2 ...αn = t 2 (α1 )s2 (α1 ) · t 2 (α2 )s2 (α2 ) · · · t 2 (αn )s2 (αn )
n


bình phương
của 1 số hữu tỉ. Tuy nhiên, f (0) = (−1) α α
1 2 ...α n nên f (0) =
α α ...α là số nguyên.
1 2 n

Từ hai điều trên suy ra f (0) là số chính phương, mâu thuẫn.
Do đó, ta có g(x) bất khả quy.

Chú ý.
Trường hợp đặc biệt của bài toán trên có xuất hiện trong đề Nhật Bản 1999: Cho n là số
nguyên dương. Chứng minh rằng đa thức (x 2 + 12 )(x 2 + 22 ) · · · (x 2 + n2 ) + 1 bất khả quy.
Nhiều bạn có sự nhầm lẫn rằng nếu f (x) bất khả quy thì các đa thức f (x 2 ), f (x 3 ), ... cũng
thế. Điều này không đúng vì chẳng hạn: f (x) = x 2 + x + 1 bất khả quy nhưng

f (x 2 ) = x 4 + x 2 + 1 = (x 2 + x + 1)(x 2 − x + 1),

là đa thức khả quy.


Trên thực tế, tiêu chuẩn Capelli đã giải quyết vấn đề này liên quan đến tính bất khả quy của
các hàm hợp, bạn đọc có thể tham khảo thêm tại [10]. Phát biểu tiêu chí này liên quan đến
các khái niệm không sơ cấp là trường (có thể hiểu là tập hợp và các phép toán được định
nghĩa trên đó) như sau: Cho K là một trường con của C và f , g ∈ K[x]. Gọi α là nghiệm phức
của f và giả sử f bất khả quy trên K. Biết rằng g(x) − α bất khả quy trên K[α][x]. Khi đó
f (g(x)) là đa thức bất khả quy trên K[x].
Tuy nhiên, nhờ vào tiêu chí đó, có thể chứng minh các bài khá đặc trưng:
1) Cho Æ
f là đa thức hệ số nguyên và p là một số hữu tỉ. Chứng minh rằng nếu f bất khả
n
quy và p
f (0) là số vô tỉ thì f (x p ) bất khả quy. 2) Chứng minh rằng đa thức (x 2 + 2) +

5(x 2n−1 + 2x n + 1) bất khả quy với mọi n ¾ 1.


Ví dụ 5. Cho số nguyên tố p ¾ 5. Tìm số các đa thức bất khả quy có dạng x p + px k +
p x l + 1 với k > l và k, l ∈ {1, 2, ..., p − 1}.

Lời giải. Ta thấy k − l phải là số chẵn vì nếu ngược lại, k − l lẻ thì do x p + 1 chia hết
cho x + 1 và px k + px l = px l (x k−l + 1) cũng chia hết cho x + 1.
Điều này cho thấy đa thức ban đầu chia hết cho x + 1, tức là nó khả quy, mâu thuẫn.
10

Đặt k = l + 2r với r ∈ Z+ và xét Q(x) = P(x − 1), rõ ràng Q(x) cũng là đa thức bất
khả quy. Ta có P(x − 1) = (x − 1) p + p(x − 1)k + p(x − 1)l + 1.
Ta thấy rằng sau khi khai triển nhị thức Newton, hệ số tự do −1, 1 bị khử mất; hai
biểu thức ở giữa khi đặt nhân tử chung p ra ngoài thì hệ số tự do của biểu thức cón
lại là 2 hoặc −2 (do k, l cùng tính chẵn lẻ). Suy ra Q(x) có dạng Q(x) = P(x − 1) =
x p + p xR(x) ± 2p với R(x) là đa thức có bậc nhỏ hơn p − 1.
Theo tiêu chuẩn Eisenstein thì đa thức này bất khả quy.
Do đó, tất cả cặp số k, l thỏa mãn là p > k > l > 0 và k − l chẵn và tổng số cặp đó là
(p − 1)(p − 3)
2C 2p−1 = .
2 4

(p−1)(p−3)
Vậy số đa thức cần tìm là 4
.
Chú ý.
Về việc đếm số đa thức bất khả quy, ta có một bài toán nổi tiếng là: Trên một trường nào đó,
tồn tại vô số đa thức bất khả quy (tương tự như có vô hạn số nguyên tố).
Ngoài ra, ta xét bài toán: Cho p là số nguyên tố. Hỏi có bao nhiêu đa thức monic có bậc là d
và hệ số từ 0 đến p − 1 và bất khả quy?

Người ta thường ký hiệu tập hợp 0, 1, 2, ..., p − 1 bởi Z p và tập hợp các đa thức lấy hệ số
trên đó là Z p [x].
d
Người ta chứng minh được tích của tất cả các đa thức như thế là x q − x nên số đa thức
P rằng d/k
1
thỏa mãn là Md (q) = d µ(k)q . Đây là số Necklace.
k|d

Liên quan đến tính khả quy của một đa thức trên tập hợp số nguyên và trên Z p , có một tiêu
n
chuẩn như sau: Cho đa thức monic P(x) =
P
ai x i có hệ số nguyên. Giả sử tồn tại số nguyên
i=0
n
tố p sao cho đa thức tương ứng Q(x) = bi x i và bi , 0 ¶ bi < p là số dư của ai khi chi cho p
P
i=0
và Q(x) bất khả quy trên Z p [x] thì P(x) bất khả quy trên Z.
Có một bài liên quan đến dạng này như sau: Với n ¾ 7 là số nguyên dương, cho đa thức
f (x) = x i−1 . Chứng minh rằng f (x) bất khả quy.
P
gcd(i,n)=1,1¶i¶n

Ví dụ 6. Cho a là số hữu tỉ và n là số nguyên dương. Chứng minh rằng P(x) =


n n
x 2 (x + a)2 + 1 là đa thức bất khả quy.
n+1
Lời giải. Với a = 0, dễ thấy P(x) = x 2 + 1 là đa thức cyclotomic nên bất khả quy.
Xét a 6= 0, giả sử P(x) khả quy. Đặt x = y − 2a thì ta có đa thức
‚ Œ2n
a2
n n
 a ‹2  a ‹2
Q( y) = y− y+ +1= 2
y − + 1.
2 2 4
a2
Đây là đa thức cyclotomic với biến z = y 2 − 4
nên rõ ràng, Q( y) không chia hết cho
đa thức nào có dạng f ( y ) - là các hàm số chẵn.
2

Tiếp theo, ta đặt


‚ Œ2n r
a2 Y
Q( y) = y2 − +1= f i ( y)mi
4 i=1
11

với f i là đa thức monic, bất khả quy. Dễ thấy Q( y)là hàm số chẵn nên ta phải có
r r
f i ( y)mi = f i (− y)mi .
Q Q
i=1 i=1

Ngoài ra, do Q( y) không chia hết cho đa thức nào có dạng f ( y 2 ) nên các f i ( y) ở trên
không phải là hàm chẵn, tức là ta phải có f i ( y) 6= f i (− y).
Do đó, với mọi i mà 1 ¶ i ¶ rthì tồn tại j 6= i sao cho f i ( y) = ± f j ( y). Từ đây suy ra r
là số chẵn và các đa thức bất khả quy ở trên có thể được chia thành 2r cặp.
Không mất tính tổng quát, có thể giả sử f1 , f2 , ..., f r là các đa thức trong các cặp khác
2

nhau và f i+ r (− y) = ± f i ( y) cùng cặp với đa thức f i . Xét tiếp đa thức


2

r/2
Y
R( y) = f i mi ( y).
i=1

Dễ thấy rằng R( y) là đa thức monic có bậc 2n và Q( y) = H( y)H(− y).


n
Đặt R( y) = y 2 + ... + b với b = R(0). So sánh hệ số tự do của 2 vế của đẳng thức trên,
€ Š2n+1 € Š2n
ta có 2a + 1 = b2 ⇔ c 4 + 1 = b2 với c = 2a .
€ Š4 2
Đặt c = uv với u, v ∈ Z và v 6= 0. Khi đó, ta được uv + 1 = b2 ⇔ u4 + v 4 = (bv 2 ) ,
suy ra bv 2 là số nguyên do u4 + v 4 nguyên. Đặt tiếp bv 2 = s ∈ Z thì ta đưa về phương
trình nghiệm nguyên u4 + v 4 = s2 .
Chú ý rằng phương trình Diophante này vô nghiệm nên điều giả sử ban đầu là sai.
Bài toán được giải quyết hoàn toàn.

Chú ý.
Với a = 1, ta có đề thi Romanian TST 1998 và Hongkong TST 2011.
Trong bài toán trên, ta có đề cập đến khái niệm
Q đa thức cyclotomic. Dưới đây là các thông tin
n
−1
chi tiết hơn: Đa thức bậc n có dạng Φn (x) = (x − ξ) = xx−1 với ξ 6= 1 là các căn bậc n của
đơn 1 (trên tập hợp số phức) được gọi là cyclotomic. Khi đó:
x n −1
Φn (x) = Φd (x)
với d là các ước nguyên dương của n. Các tính chất và bài toán liên quan: Bài
1. Chứng minh x p−1 + x p−2 + ... + x 2 + x + 1 bất khả quy với p là số nguyên tố.
Bài 2. Chứng minh rằng với mọi n thì Φn (x)bất khả quy.
Bài 3. Chứng minh P(x) = 1 + x p + x 2p + ... + x p(p−1) bất khả quy với p là số nguyên tố.
Trong đề VMO của Việt Nam 2014, có một bài toán dạng tương tự là: Cho đa thức P(x) =
2n
(x 2 − 7x + 6) + 13 với n nguyên dương. Chứng minh rằng P(x) không thể biểu diễn thành
tích của n + 1 đa thức với hệ số nguyên.
Ta thấy rõ ràng Q(x) = P(x + 1) = x 2n (x + 5)2n + 13 và lập luận theo kiểu tương tự bài toán
6, ta có thể chứng minh kết quả tổng quát hơn đề VMO là nếu Q(x) khả quy thì nó có thể
được phân tích thành tích của 2 đa thức đồng bậc.
Còn đa thức Q(x) có bất khả quy với n tùy ý hay không thì vẫn là câu hỏi mở. Ngoài ra, với
đa thức f (x) là đạo hàm của 1 đa thức cyclotonic, người ta vẫn chưa chứng minh được là nó
có bất khả quy với n tùy ý hay không.
Ví dụ 7. Cho đa thức P(x) có bậc là m không vượt quá 10. Giả sử rằng P(0) = 0 và đa
thức P(x) có m nghiệm nguyên phân biệt. Biết rằng P(x) + 1 khả quy. Tìm tất cả các
giá trị có thể có của P(2).
12

Lời giải. Đặt P(x) = Q 1 (x)Q 2 (x) − 1 với Q 1 (x), Q 2 (x) là các đa thức hệ số nguyên
có bậc dương. Giả sử các nghiệm của P(x) là r1 , r2 , ..., rm thì rõ ràng Q 1 (ri )Q 2 (ri ) = 1
với mọi i = 1, m. Tuy nhiên, do Q 1 (x), Q 2 (x) có hệ số nguyên nên giá trị của chúng
tại ri chỉ có thể cùng là 1 hoặc cùng là −1, tức là Q 1 (x) = Q 2 (x) tại m điểm hay
Q 1 (x) − Q 2 (x) là đa thức bậc nhỏ hơn m nhưng lại triệt tiêu tại m điểm. Suy ra
Q 1 (x) = Q 2 (x).
Do đó P(x) = Q2 (x) − 1 và dễ thấy m là số chẵn. Suy ra Q(x) bằng 1 tại đúng m
2
điểm
và bằng −1 tại đúng m2 điểm, có thể viết là

Q(n) = (n − r1 )(n − r2 ) · · · (n − r m ) − 1 = (n − r m +1 )(n − r m +2 ) · · · (n − rm ) + 1.


2 2 2

Từ đây, dễ dàng chứng minh được Q(x) có bậc không vượt quá 2.
Nếu Q(x) có bậc 1 thì Q(x) = x + 1 hay Q(x) = x − 1 (chú ý rằng P(0) = 0 nên hệ số
tự do của Q(x) chỉ có thể là 1 hoặc −1).
Nếu Q(x) có bậc 2 thì Q(x) = x 2 +ax + b và chú ý rằng hai đa thức sau đều có nghiệm
nguyên: x 2 + ax + b + 1 và x 2 + ax + b − 1.
Suy ra a2 − 4(b + 1), a2 − 4(b − 1) đều phải là số chính phương. Do đó a2 − 4b = 5
và chú ý rằng b = ±1 nên nên có 4 đa thức tương ứng thỏa mãn là x 2 − 3x + 1, x 2 +
3x + 1, x 2 − x − 1, x 2 + x − 1.
Vậy P(2) = Q2 (2) − 1 có thể nhận các giá trị là 0, 8, 24, 120.

3. Về một số tiêu chuẩn bất khả quy khác.


3.1. Về các đa thức nhận 1 giá trị tại nhiều điểm phân biệt.
Ta xét 2 bài toán quen thuộc sau:
Bài 1. Chứng minh rằng đa thức P(x) = (x − a1 )(x − a2 )(x − a3 )...(x − an ) − 1 bất
khả quy với a1 , a2 , ..., an là các số nguyên phân biệt.
Bài 2. Cho đa thứcP(x) = (x − a1 )(x − a2 )...(x − an ) + 1. Hỏi với các giá trị nào của
a1 , a2 , ..., an và n thì đa thức trên khả quy?
Các bài này giải quyết cũng khá nhẹ nhàng như sau: với bài 1, nếu P(x) = f (x) · g(x)
thì dễ thấy f (ai ) · g(ai ) = −1 và dẫn đến f (ai ) + g(ai ) = 1 với mọi i = 1, n, từ
đây dễ thấy điều vô lý. Còn với bài 2, ta chứng minh nếu có P(x) = f (x) · g(x) thì
f (x) = g(x), từ đó xét các trường hợp để đi đến kết luận chỉ có 2 lớp đa thức thỏa
mãn (với a ∈ Z) là:

• P(x) = (x − a1 )(x − a2 ) + 1 = (x − a)(x − a − 1) + 1 = (x − a − 1)2 và


2
• P(x) = (x−a)(x−a+1)(x−a−1)(x−a−2)+1 = (x 2 − (2a − 1)x + a2 + a − 1) .

Bằng cách tương tự, ta có thể giải quyết bài toán sau:
Bài 3. Chứng minh rằng với mọi bộ số nguyên a1 , a2 , · · · , an đôi một khác nhau thì
đa thức P(x) = (x − a1 )2 (x − a2 )2 . . . (x − an )2 + 1 cũng bất khả quy.
deg P(x)
Bài 4. Cho đa thức P(x) có hệ số nguyên và tồn tại k > 2 số nguyên a1 , a2 , · · · , ak

thỏa mãn P(ai ) = 1 với i = 1, k. Chứng minh rằng P(x) bất khả quy.
13

Bạn đọc có thể tham khảo chi tiết thêm tại [1] hoặc [8]. Ngoài ra, người ta còn có các
nghiên cứu mở rộng rất nhiều:
[Wegner] Đa thức (x − a1 )4 (x − a2 )4 · · · (x − an )4 + d với n > 5 và d > 0, d 6 ≡3 (mod
4) thì bất khả quy.
[Dorwart and Ore] Cho đa thức f (x) = b0 x 2 + b1 x +1, P(x) = (x −a1 )(x −a2 ) . . . (x −
an ), n ¾ 5 thì f (P(x)) bất khả quy.
Các đa thức như thế được gọi là Polynomial of Schur-type and Polya-type và người ta
– ™2 k
n
(x − ai ) + d.
Q
còn qua tâm đến các tiêu chuẩn khả quy cho đa thức có dạng
i=1

3.2. Tiêu chuẩn Cohn.


k
P
Cho p là số nguyên tố và biểu diễn của p trong hệ thập phân là ai 10i với 0 ¶ ai ¶ 9
i=0
n
và 0 ¶ i ¶ k. Khi đó, đa thức tương ứng f (x) =
P
ai x i bất khả quy.
i=0

Người ta đã tổng quát được khi thay hệ thập phân bởi một hệ cơ số bất kỳ. Trong
trường hợp hệ nhị phân, ta có phát biểu khá đẹp mắt như sau:

Cho đa thức f (x) có các hệ số đều là 0 hoặc 1 và f (2) là số nguyên tố. Chứng minh
rằng f (x) bất khả quy.
Chứng minh chi tiết và các mở rộng của tiêu chuẩn này có thể tham khảo thêm tại
[7], [9] hoặc [10].
3.3. Tiêu chuẩn Dumas.
Cho đa thức f (x) = a0 x n + a1 x n−1 + · · · + an là đa thức có hệ số nguyên. Giả sử tồn tại
số nguyên tố p sao cho nếu đặt ri = vp (ai ), 0 < i ¶ n (quy ước ri = +∞ nếu ai = 0)
r r
thì r0 = 0, ii > nn với 1 ¶ i ¶ n − 1 và gcd(rn , n) = 1. Khi đó, f (x) bất khả quy.
Ta thấy tiêu chuẩn Eisenstein là trường hợp đặc biệt của định lý trên với rn = 1. Người
ta cũng đã mở rộng các tiêu chuẩn này ra rất nhiều nhưng nó nằm ngoài phạm vi của
bài viết này, bạn đọc có thể tham khảo thêm tại [6].
3.4. Về tiêu chuẩn bất khả quy cho đa thức nhiều biến.
Đa thức 1 biến có bậc lớn hơn 1 trên tập hợp số phức luôn khả quy. Nhưng với đa thức
nhiều biến thì điều đó không còn đúng nữa. Do đó, khi nói đến tính khả quy của đa
thức nhiều biến, ta cần chỉ rõ là xét trên tập hợp nào.
Chẳng hạn, đa thức x 2 + y 2 là bất khả quy trên tập hợp số nguyên nhưng lại khả quy
trên tập hợp số phức x 2 + y 2 = (x + i y)(x − i y).
Đa thức x 2 + y 2 − 1 là bất khả quy trên cả tập số phức và còn được gọi là bất khả quy
tuyệt đối (absolutely polynomial).
Trên thực tế, không có nhiều tiêu chuẩn về tính bất khả quy cho đa thức nhiều biến
nên hầu hết các bài toán dạng này, ta cần phải lập luận trực tiếp mọi thứ. Chẳng hạn
trong bài VMO 2012 có 1 bài như thế, nhưng đa thức này chỉ bất khả quy trên R và
không bất khả quy tuyệt đối:
Cho n là số nguyên dương và P(x, y) = x n + x y + y n . Chứng minh rằng không tồn
tại được G(x, y) và H(x, y) với hệ số thực sao cho P(x, y) = G(x, y) · H(x, y).

(Việt Nam 2012)


14

Một bài khác rất thú vị trong đề ELMO 2014:


Tìm tất cả n ¾ 2 sao cho đa thức P(a1 , a2 , ..., an ) = a1n + a2n + · · · + ann − na1 a2 · · · an bất
khả quy tuyệt đối. Câu trả lời là mọi n ¾ 4 đều thỏa.

3. Các bài toán tự luyện.


Bài 1.
1) Chứng minh rằng các đa thức sau bất khả quy

a) f (x) = x 4 + 5x 3 − 2x 2 − 6x + 3.

b) g(x) = 3x 4 + 7x 3 − 4x 2 + 12x + 9.

c) h(x) = 2x 4 + 13x 3 + 39x 2 + 58x + 20.


2 2
2) Chứng minh đa thức sau đây bất khả quy (x 3 + 7x − 1) − 18(x 2 + 1) .
Bài 2.
Chứng minh các đa thức sau bất khả quy:

a) P(x) = x p − px + (2p − 1),ở đây p là số nguyên tố.

b) P(x) = x p + px p−1 + px + (3p + 1) ở đây p là số nguyên tố.

c) Cho p là số nguyên tố ,k > 0và r là những số nguyên vàφ(x) là đa thức hệ số


nguyên bất kì bậc nhỏ hơn p k−1 . Chứng minh rằng nếu r không chia hết cho p,
k
đa thức P(x) = x p + pxφ(x) + (r p − 1) không phân tích được.

Bài 3. Chứng minh các đa thức sau bất khả quy:

xn n−1
x2
a) Đa thức P(x) = n!
+ (n−1)!
x
+ ... + 2!
+ 1!x + 1.

b) Đa thức x n + x 2 + x + p với n ¾ 2 và p nguyên tố.

c) Đa thức x 3 + ax 2 + bx + c bất khả quy với b chẵn và a, c lẻ.

d) Đa thức x 105 − 9.

e) Đa thức(x 2 + 1)(x 2 + 2)...(x 2 + n) + 1 bất khả quy.

Bài 4.

a) Chứng minh rằng nếu a là số nguyên không chia hết cho 5 thì đa thức P(x) =
x 5 − x + a bất khả quy.

b) Cho đa thức f (x) = x 2 + 100x + a. Chứng minh rằng tồn tại vô hạn số a sao
cho f (x) bất khả quy và cũng tồn tại vô hạn số a sao cho f (x) khả quy.

c) Cho p, q là 2 số nguyên tố. Tìm tất cả các số nguyên a sao cho P(x) = x n +
a x n−1 + pq bất khả quy.
15

d) Cho p là số nguyên tố và m, n, a là các số nguyên sao cho p + 1 < a. Chứng


minh rằng P(x) = x m (x − a)n + p bất khả quy.

e) Cho các số nguyên a1 , a2 , a3 , b1 , b2 , b3 sao cho tổng của hai đa thức bất kỳ trong
ba đa thức f i (x) = x 2 + ai x + bi với i = 1, 2, 3 đều bất khả quy trên tập số thực.
Hỏi tổng của ba đa thức này có bất khả quy trên tập số thực không?

Bài 5.

a) Chứng minh rằng nếu P(x) bất khả quy thì nó không có nghiệm bội, tức là
không tồn tại số thực a sao cho P(x) = (x − a)k Q(x) với Q(x) là đa thức nào
đó và k ¾ 2.

b) Chứng minh rằng với mọi đa thức P(x) có hệ số nguyên, tồn tại số nguyên
dương k sao cho Q(x) = 2k P(x) − 1 bất khả quy.

Bài 6. Cho số nguyên tố p và n < 2p. Xét đa thức P(x) = x 2p + px n − 1. Chứng minh
rằng:

a) Nếu p = 2 thì P(x) bất khả quy.

b) Nếu như tồn tại 2 đa thức hệ số nguyên f (x), g(x) sao cho f (x)g(x) = P(x) thì 
cũng tồn tại hai số nguyên u, v không đồng thời bằng 0 sao cho deg u f (x) + v g(x) =
0.

c) Tồn tại A(x), B(x) là hai đa thức có bậc không vượt quá p − 2 sao cho f (x) =
x p + pxA(x) − 1, g(x) = x p + px B(x) + 1.

d) Đa thức P(x) bất khả quy với mọi số nguyên tố p > 2.


n
Bài 7. Cho N là số nguyên dương. Xét đa thức f (x) = ai x i là đa thức hệ số nguyên
P

i=0
với ai ¶ N với i = 0, n. Chứng minh rằng nếu tồn tại ít nhất 2N + 4 số nguyên sao
cho giá trị tuyệt đối của f (x) tại các số nguyên đó là số nguyên tố thì f (x) bất khả
quy.
Bài 8. a) (Romanian TST 1997) Cho P(x), Q(x) là các đa thức monic, bất khả quy
và có hệ số hữu tỉ. Giả sử P(x), Q(x) lần lượt có nghiệm là α, β với α + β là số hữu tỉ.
Chứng minh rằng đa thức P 2 (x) − Q2 (x) cũng có nghiệm hữu tỉ.
b) Câu hỏi còn đúng không nếu thay P 2 (x) − Q2 (x) bởi P(P(x)) − Q(Q(x))?
Bài 9. a) Chứng minh rằng đa thức x y 3 + x 2 y 2 − x 5 y + x 2 + 1 không thể phân tích
được thành tích của hai đa thức hai biến f (x, y) và g(x, y).
b) (ELMO 2012 Shortlist) Chứng minh rằng nếu m, n nguyên tố cùng nhau thì không
tồn tại hai đa thức f (x, y) và g(x, y) có hệ số thực thỏa mãn f (x, y) · g(x, y) =
x m − y n.
c) (ELMO 2012 Shortlist) Cho f , g là các đa thức thỏa mãn gcd(deg f , deg g) = 1.
Giả sử tồn tại các đa thức P(x, y), Q(x, y) với hệ số phức sao cho f (x) + g( y) =
P(x, y) · Q(x, y). Chứng minh rằng P, Q là đa thức hằng.
Bài 10. a) Với n ¾ 0, xét dãy đa thức Pn (x) xác định bởi

P0 (x) = 1, P1 (x) = x − 1 và Pn (x) = x Pn−1 (x) − Pn−2 (x)


16

với n ¾ 2. Xác định tất cả số nguyên dương n sao cho P(x) khả quy trên tập hợp số
hữu tỉ.
b) (China TST 2008) Chứng minh rằng với mọi n ¾ 2, tồn tại đa thức f (x) = x n +
a1 x n−1 + a2 x n−2 + · · · + an thỏa mãn đồng thời các điều kiện sau:
i. Các hệ số a1 , a2 , ..., an đều khác 0.

ii. Với mọi số tự nhiên x thì f (x) không phải là số nguyên tố.

iii. f (x) bất khả quy.


Bài 11. 1) (Iran 3th round 2011) Cho f (x) là đa thức monic có bậc 2 với hệ số
nguyên sao cho f (x) > 0, ∀x và f (0) là số square-free khác ±1. Chứng minh rằng với
mọi số tự nhiên n thì f (x n ) là đa thức bất khả quy.
2) Cho đa thức f (x) bậc lẻ, bất khả quy và có hệ số nguyên. Biết rằng module của
tất cả các nghiệm của đa thức đều lớn hơn 1 và f (0) là số square-free. Chứng minh
rằng f (x 3 ) là đa thức bất khả quy.
Bài 12. 1) (Mathlinks context) Chứng minh rằng với a là số nguyên khác 0 và n ¾ 3
thì đa thức sau bất khả quy: x n + a(x n−1 + x n−2 + ... + x 2 + x) − 1.
2) Cho a1 ¾ a2 ¾ ... ¾ an > 0 là các số nguyên. Chứng minh đa thức P(x) = x n −
Pn
ai x n−i là đa thức bất khả quy.
i=1

Bài 13. 1) Cho (b0 , b1 , b2 , b3 ) là hoán vị của {54, 72, 36, 108}. Hỏi x 5 + b3 x 3 + b2 x 2 +
b1 x + b0 − 1 có luôn bất khả quy hay không?
2) (Iran National Olympic 2008) Cho (b0 , b1 , b2 , b3 ) là hoán vị của {54, 72, 36, 108}.
Chứng minh x 5 + b3 x 3 + b2 x 2 + b1 x + b0 bất khả quy.
Bài 14. 1) Cho a1 , a2 , ..., an là các nguyên đôi một khác nhau. Xét đa thức

P(x) = (x − a1 )(x − a2 )...(x − an ) − 2.

Biết rằng P(x)có thể được biểu diễn dưới dạng tích của hai đa thức với hệ số nguyên
và có bậc ¾ 1. Chứng minh rằng deg P(x) = 3.
2) Giả sử n nghiệm của đa thức bậc n với các hệ số hữu tỉ tạo thành 1 cấp số cộng.
Chứng minh rằng nếu đa thức này bất khả quy thì bậc của nó không lớn hơn 2.
n
3) Cho đa thức f (x) =
P
ai x i và tồn tại số nguyên tố p thỏa mãn điều kiện
i=0

. . .
an−2 , an−3 , ..., a0 ..p và an 6 ..p, a0 6 ..p2 .

Chứng minh rằng nếu f (x)khả quy thì nó cũng có nghiệm hữu tỉ.
Bài 15.
1) (Iran MO 2007) Chứng minh rằng với mỗi tập hợp khác rỗng S ⊂ N, |S| ¾ 2 khi
và chỉ khi tồn tại một dãy a0 , a1 , ..., an trong S sao cho với mỗi n nguyên dương
n
thì đa thức f (x) =
P
ai x i bất khả quy.
i=0

2) (Iran MO 2007) Có tồn tại hay không dãy số tự nhiên a0 , a1 , a2 , ... sao cho với
n
mỗi i 6= j thì gcd(ai , a j ) = 1 và với mỗi n, đa thức f (x) =
P
ai x i bất khả quy?
i=0
17

Bài 16.
1) Cho p là một số nguyên tố và klà một số tự nhiên. Chứng minh rằng P(x) =
k
x p −1
k không phân tích được.
x p −1 −1
x a −ax+a−1
2) Cho a ¾ 3 là một số square-free. Chứng minh rằng f (x) = (x−1)2
bất khả
quy.

TÀI LIỆU THAM KHẢO

1. Nguyễn Văn Mậu, Biến phức – Định lý và áp dụng, 2009.

2. Trần Nam Dũng, Đa thức.


3. http://www.artofproblemsolving.co
4. http://mathoverflow.net/
5. http://www.imc-math.org.uk/
6. Sudesh K. Khanduja, Some extensions and applications of Eisenstein Irreducibil-
ity Criterion.

7. John Brillhart, Michael Filaseta, On an irreducibility theorem of A. Cohn, 1981.

8. H. L. Dorwart and Oystein Ore, Criteria for the Irreducibility of Polynomials,


1933

9. Yefei Hao, Integer Polynomial, 2007.


10. Titu Andresscu, Problem from the book, 2007.
11. Louis Weisner, Irreducibility of polynomials of degree n which assume the same
value n times. A.Borisov, M.Filaseta, T.Y.Lam, O.Trifonov, Classes of polynomials
having only one non-cyclotomic irreducible factor.
12. [13] Steven H.Weintraub, Several proofs of the irreducibility of the cyclotomic
polynomials. K. Gyory, L. Hajdu, rreducibility criteria of Schur-type and Polya-
type.
13. [15] Wilhelm Ljunggren, On the irreducibility of certain trinomials and quadri-
nomials.
14. Ernst S.Selmer, On the irreducibility of certain trinomials.
15. Nguyễn Công Hậu, Võ Thị Mai Phương, Nguyễn Huỳnh Như Ý, Nguyễn Cao Tín,
Phạm Ngọc Luân, Đa thức và ứng dụng, 2012.

You might also like